Induction (1 Viewer)

Makematics

Well-Known Member
Joined
Mar 26, 2013
Messages
1,829
Location
Sydney
Gender
Male
HSC
2013
Hi guys, I need help with an induction.

Use mathematical induction to prove that for all integers n≥1,

√1 + √2 + √3 + ... + √n ≤ [(4n+3)√n] / 6

So far I have done the easy bit...

Step 1: Prove true for n=1
LHS=1
RHS= 7/6
LHS≤RHS
Hence it is true for n=1.

Step 2: Assume true for n=k
i.e. Assume √1 + √2 + √3 + ... + √k ≤ [(4k+3)√k] / 6

Step 3: Prove true for n=k+1
i.e. Prove √1 + √2 + √3 + ... + √k + √(k+1) ≤ [(4k+7)√(k+1)] / 6

(BTW Those symbols are square roots)
 

braintic

Well-Known Member
Joined
Jan 20, 2011
Messages
2,137
Gender
Undisclosed
HSC
N/A
Hi guys, I need help with an induction.

Use mathematical induction to prove that for all integers n≥1,

√1 + √2 + √3 + ... + √n ≤ [(4n+3)√n] / 6

So far I have done the easy bit...

Step 1: Prove true for n=1
LHS=1
RHS= 7/6
LHS≤RHS
Hence it is true for n=1.

Step 2: Assume true for n=k
i.e. Assume √1 + √2 + √3 + ... + √k ≤ [(4k+3)√k] / 6

Step 3: Prove true for n=k+1
i.e. Prove √1 + √2 + √3 + ... + √k + √(k+1) ≤ [(4k+7)√(k+1)] / 6

(BTW Those symbols are square roots)
This is Exercise 8.1 Q7a from Terry Lee. You can look at the solutions at the back of his book.
 

Sy123

This too shall pass
Joined
Nov 6, 2011
Messages
3,730
Gender
Male
HSC
2013
Hi guys, I need help with an induction.

Use mathematical induction to prove that for all integers n≥1,

√1 + √2 + √3 + ... + √n ≤ [(4n+3)√n] / 6

So far I have done the easy bit...

Step 1: Prove true for n=1
LHS=1
RHS= 7/6
LHS≤RHS
Hence it is true for n=1.

Step 2: Assume true for n=k
i.e. Assume √1 + √2 + √3 + ... + √k ≤ [(4k+3)√k] / 6

Step 3: Prove true for n=k+1
i.e. Prove √1 + √2 + √3 + ... + √k + √(k+1) ≤ [(4k+7)√(k+1)] / 6

(BTW Those symbols are square roots)




















Okay, now this is quite a good question because the inequality is very.... 'accurate', as in the bounds are quite close. Note that if the question was designed so that Step 3 ended up with (4k+9) as the bracket, teh question is simple as we simply acknowledge that sqrt(k) < sqrt(k+1)

But the solution I have, is very much derived from the answer. What I did was.
If the expression on the RHS of what we need is correct, then the following inequality holds:



This is true IF Step 3 is true. So that means the problem resolves itself into proving the above inequality. The LHS is from adding sqrt(k+1) to Step 2.

That means:

IF Step 3 is true, then:



Then expanding, then simplifying, will yield, the NEED to prove that:



Now, from the AM-GM inequality:



Setting a and b appropriately:



That means that half of 4k+1 is greater than the RHS, that means 1 of 4k+1 is greater than RHS



Technicalities.

That means, inequality: must hold.

Therefore, we can establish what needed to be true:



Hence, Step 3 is true.

Therefore:

*Induction essay*

Proven by mathematical induction
 

Trebla

Administrator
Administrator
Joined
Feb 16, 2005
Messages
8,132
Gender
Male
HSC
2006
First result we need is that for positive integer k



Proof: First note that for positive integer k



and the result follows.

Now to the inductive step:

 

Makematics

Well-Known Member
Joined
Mar 26, 2013
Messages
1,829
Location
Sydney
Gender
Male
HSC
2013
Ah thanks heaps guys :D @Braintic ah right i got it from cambridge, was unaware that it was also in Terry Lee.
@Sy and @Trebla : Cheers :) Nice solutions.
 

Makematics

Well-Known Member
Joined
Mar 26, 2013
Messages
1,829
Location
Sydney
Gender
Male
HSC
2013
hmm i did this exact method, but my dad told me that the above statement was incorrect :/ i agree that 1 is greater than or equal to 0, although i would like a clear explanation of why it is so. could you explain why it is correct?
 
Last edited:

Trebla

Administrator
Administrator
Joined
Feb 16, 2005
Messages
8,132
Gender
Male
HSC
2006
hmm i did this exact method, but my dad told me that the above statement was incorrect :/ i agree that 1 is greater than or equal to 0, although i would like a clear explanation of why it is so. could you explain why it is correct?
1 is either {greater than zero} OR {equal to zero}.

In this case we know with certainty that the former is true in a strict sense (1 > 0 is a strict inequality), but the statement of the weak inequality is still true.
 
Last edited:

Makematics

Well-Known Member
Joined
Mar 26, 2013
Messages
1,829
Location
Sydney
Gender
Male
HSC
2013
hmm yeah that makes sense i guess. Also trebla how do you know that you have to start off with 1 is greater than or equal to 0?
 

Trebla

Administrator
Administrator
Joined
Feb 16, 2005
Messages
8,132
Gender
Male
HSC
2006
hmm yeah that makes sense i guess. Also trebla how do you know that you have to start off with 1 is greater than or equal to 0?
Work backwards from the inequality you need to prove
 

Users Who Are Viewing This Thread (Users: 0, Guests: 1)

Top